Feb 22, 2011

PREDICTOR TEST 5

PREDICTOR TEST 5



Integumentary/EENT: 5 items
Gastrointestinal: 10 items
Endocrine: 5 items
Fundamentals: 10 items
Fluid and Electrolytes: 5 items
Psych: 15 items
Musculoskeletal: 10 items
Cardiovascular: 10 items
Respiratory: 5 items
Infectious/ Precaution: 15 items
Neurologic: 10 items
MCN/G&D: 11 items
Pharmacology: 20 items
Delegation: 5 items
Total: 136 items


Integumentary/ EENT: 5 items

1. The nurse is observing a newly graduate nurse caring for a burn patient who sustained a 3rd degree burn in trunk and leg during a house fire .The nurse intervenes if the new nurse performed which of the following?
A. The nurse administers 100% oxygen via a tight-fitting, non-rebreather face mask
B. The nurse administers pain medication through IM route
C. The nurse elevates the circumferential burns of the extremities on pillows
D. The nurse places a bed cradle on the patient’s bed
Rationale: B. IM and SQ routes of administering medication should be avoided because absorption through the soft tissue is unreliable when hypovolemia and large fluid shifts occur. (Saunders Comprehensive Review, 3rd ed. © 2006, p. 547)

2. A nurse is administering fluid intravenously to a client who sustained a massive 3rd degree burn in the body. In evaluating the adequacy of fluid replacement, the nurse would consider which of the following parameters as a good indicator of effective fluid replacement?
A. urine output of 50 ml for 2 hours
B. BUN of 45 mg/dL
C. Urine specific gravity of 1.040
D. Apical heart rate of 98 bpm
Rationale: D. Successful or adequate fluid resuscitation in the adult is signaled by stable vital signs, adequate urine output, palpable peripheral pulses, and clear sensorium. For an adult, hourly urine volume should be 30-50 mL. a BUN level of 45 mg/dL ( NV: 8-25 mg/dL) indicates an elevated level and signifies a slowing of glomerular filtration rate secondary to decreased tissue perfusion. Normal urine specific gravity ranges from 1.016-1.022. very concentrated urine has a high specific gravity that indicates a possible dehydration. Apical heart rate of 98 bpm is considered within normal limits (60-100 bpm) and shows a good condition of circulating blood volume.
(MS by Black and Hawks, 7th ed. © 2005, p. 208);
(Saunders Comprehensive Review, 3rd ed. © 2006, p. 118, 546)

3. A comatose client is at risk for the development of pressure ulcers; therefore, the nurse should include which of the following activity in the plan of care?
A. Establishing an individualized turning schedule
B. Massaging red areas on bony prominences regularly
C. Use of donut-shaped cushion on the sacral area
D. Washing skin with soap and water frequently
Rationale: A. Turning and positioning interventions, tailored to the client’s needs, are designed to reduce the pressure and shearing force in the skin. Massage applied directly to the reddened area further promotes skin breakdown. Soap and water causes drying of the skin which promotes skin breakdown. A donut-shaped cushion interferes with the oxygenation of the tissue of the sacrum in the center of the donut, thus contributing to decubitus formation. . (Cracking the NCLEX-RN by J. Meyer, 7th ed. © 2003, p. 198)

4. A 24-year-old diver has been diagnosed of acute otitis media. Which of the following procedures is the most common surgical intervention for him?
A. Tympanoplasty
B. Myringotomy
C. Ossiculoplasty
D. Mastoidectomy
Rationale: B. Myringotomy involves making an incision in the tympanic membrane to relieve pressure and drain fluid from the ear. Tympanoplasty involves surgical reconstruction of the tympanic membrane and is done to reestablish middle ear function, close perforations, and prevent recurrent infection. It is commonly performed in chronic otitis media. (Lippincott’s Review for NCLEX-RN, 8th ed., p. 516)

5. A nurse is providing discharge teaching to a client after stapedectomy. All of the following comprises the nurse’s instructions, except
A. Weightlifting can be resume after 3 weeks
B. Avoiding air travel for at least 1 week
C. Avoiding swimming for at least 3 weeks
D. Opening the mouth while coughing and sneezing
Rationale: C. Post stapedectomy, heavy lifting should be strictly avoided for at least 3 weeks. Water in the ear and air travel should be avoided for at least 1 week. Coughing and sneezing should be performed with the mouth open to prevent increased pressure. (Mosby’s Prioritization, Delegation, and Assignment © 2006, p. 186)

Gastrointestinal: 10 items

1. Mr. G., age 67 is admitted to the hospital with ascites and jaundice and diagnosed with cirrhosis of the liver. The nurse suspects that Mr.G develops hepatic encephalopathy when she noticed which clinical manifestation in the patient?
A. Increased urine output
B. Decreased tendon reflexes
C. Altered attention span
D. Hypotension
Rationale: C. Hepatic encephalopathy commonly causes altered attention span, as well as tremors, changes in affect, irritability, and lack of coordination followed by disorientation to time, then to place. DTR usually increases with hepatic encephalopathy until the patient losses consciousness. Hypotension rarely occurs with this condition. (MS by Black and Hawks © 2005, p.1356)

2. A 40-year-old banker is admitted in the medical unit with a tentative medical diagnosis of hepatitis A. the physician determines that the patient is in the icteric stage of hepatitis A infection. Which nursing intervention is not appropriate at this stage?
A. Administering vit. K as ordered
B. Providing mittens for the patient
C. Forcing fluids (3,000 ml or more daily)
D. Encouraging ambulation
Rationale: D. During the icteric phase (after jaundice appears), the patient with hepatitis A should remain on bed rest to promote regeneration of liver tissue; physical activity should be restricted until the patient’s condition improved. The e nurse should force fluids to replace fluid loss from vomiting. The physician may order vit. K administration to aid in prothrombin formation and to prevent hemorrhage. The patient may experience pruritus during this stage; the nurse should provide mittens to prevent abrasions from the patient’s fingernails. (MS by Black and Hawks © 2005, p. 1330)

3. As the patient receive discharge instructions, which of the following should the nurse stressed to the patient?
A. “Wear medical alert bracelet at all times”
B. “Use a condom during sexual intercourse”
C. “emphasize needle precautions”
D. “Never donate blood”
Rationale: D. before discharge, the nurse should caution the patient with any type of hepatitis against ever donating blood. The virus may remain active in the serum long after the acute illness passes; donating blood could transmit the disease to the recipient. Hepatitis A is not transmitted through sexual contact; therefore the use of condom during sexual intercourse should not be the focused of the discharge instructions. (Saunders Comprehensive Review © 2006, p. 692)

4. A 43-year-old construction worker is about to be discharged after being admitted because of pancreatitis. The nurse’s discharge planning includes dietary teaching. Which instructions would not be appropriate for the patient?
A. Avoid taking antacids
B. Avoid coffee
C. Avoid large meals
D. Avoid spicy foods
Rationale: A. Necessary long-term dietary changes would include avoiding rich and spicy foods, heavy meals, alcohol, and coffee as well as using antacids to decrease the production of gastric acid and neutralize gastric pH, which stimulates the pancreas. Antacids are used after the acute stage of pancreatitis has passed. (Saunders Comprehensive Review © 2006, p. 691)

5. Mr. Y., age 56, was admitted to the hospital with chronic gastroenteritis. To meet Mr.Y’s nutritional needs, the physician orders total parenteral nutrition (TPN) via central venous catheter. Which guideline should the nurse remember when choosing food for Mr.Y’s menu when he is taken off in TPN therapy?
A. Low-protein food should be introduced slowly to reduce damage to the renal membrane
B. Foods selected should be chemically and mechanically nonirritating and high in protein
C. Certain fluids, such as cocoa and carbonated drinks, tends to increase upper jejunal digestion and must be included in the diet plan
D. Keeping in mind the patient’s like and dislikes, foods selected should be high in residue to stimulate motility
Rationale: B. The nurse should select foods that are chemically and mechanically nonirritating and are high in protein to restore normal nutritional levels. All foods should be nutritionally balanced, residue-free so that they can be digested in the jejunum. All foods should be introduced slowly and they should be high in protein. Certain fluids, such as cocoa and carbonated drinks, should be excluded because they tend to irritate the bowel. (Nurse Test: Med-Surg.,p. 121)

6. A nurse develops a teaching plan for a client diagnosed with hepatitis B. Which diet, when selected by the client, would indicate that teaching has been effective?
A. Carrots, lean beef, and orange juice
B. Bacon, eggs, milk
C. Shrimp, avocado salad, and skim milk
D. Hamburger, cottage cheese, and malted milk
Rationale: A. A client with hepatitis should be encouraged to have a low-fat, cholesterol restricted diet. (MS by Black and Hawks © 2005, p. 1329)

7. Mr. C, a post-op patient who undergoes Billroth II procedure was assigned to your care. As you review the postoperative orders for him, which of the following, if prescribed, would the nurse question and verify?
a. Exercising the extremities
b. Assistng in breathing techniques
c. Encouraging early ambulation
d. Irrigation of the NGT

RATIONALE: D. In Billroth II procedure, the proximal remnant of the stomach is anastomosed to the proximal jejunum. Patency of the nasogastric tube is critical for preventing the retention of gastric secretions. The nurse should never irrigate or reposition the gastric tube after gastric surgery, unless specifically ordered by the physician. In this situation the nurse should clarify the order. The nurse can only assist the physician with nasogastric irrigation or removal of the nasogastric tube. Options A,B, & D are appropriate postoperative interventions. (p. 696, 702, Saunder’s, © 2006)

8. Mr. R., had a colostomy attached two days ago. The nurse notes that the client is passing a malodorous flatus from the stoma. The nurse would perform which of the following?
a. Document the observation since it is expected and considered normal
b. The nurse should attached again have the nasogastric tube removed
c. The nurse must inform the physician because the client maybe experiencing an early signs of ischemic bowel
d. This indicates inadequate preoperative bowel preparation and requires another irrigation.
RATIONALE: A. As peristalsis returns following creation of a colostomy, the client begins to pass malodorous flatus. This indicates returning bowel function and is an expected event. Within 72 hours of surgery, the client should begin passing stool via the colostomy. Options B-D are incorrect. (p. 697, Saunder’s, © 2005)

9. The nurse is providing discharge instructions to a client following Gastrectomy. Which measures will the nurse instruct the client about?
a. To prevent dumping syndrome, eat high carbohydrate foods. Abdominal cramping and pain are early signs and symptoms of dumping syndrome.
b. To prevent dumping syndrome, limit the fluids taken with meals. Sweating and pallor are early signs and symptoms of dumping syndrome.
c. To prevent dumping syndrome, ambulate following a meal. Bradycardia and indigestion are early signs and symptoms of dumping syndrome.
d. To prevent dumping syndrome, sit in high Fowler’s position during meals. Double vision and chest pain are early signs and symptoms of dumping syndrome.
RATIONALE: B. The nurse should instruct the client to decrease the amount of fluid taken at meals and to avoid high carbohydrate foods including fluids such as fruit nectars; to assume a low Fowler’s position during meals; to lie down for 30 minutes after eating to delay gastric emptying; and to take antispasmotics as prescribed.
Early manifestations of dumping syndrome occur 5-30 minutes after eating. Symptoms include vertigo, tachycardia, syncope, sweating, pallor, palpitations, and the desire to lie down. (p. 696, 702, Saunder’s, © 2005)

10. A 45-year-old office manager comes to the ED and was later diagnosed to have PUD. Upon knowing the diagnosis, the nurse must first consider which of the following action?
a. Inserting a nasogastric tube for lavage
b. Administration of antiemetics for vomiting
c. Administration of pain medication/analgesics
d. Obtaining a serum electrolyte profile

RATIONALE: C. Pain management related to epigastric burning is a priority in the patient with PUD without perforation.
A. Although insertion of a nasogastric tube may be indicated during patient assessment, it is not the initial action of the ED nurse. If a patient might require surgery, nasogastric tube insertion would be indicated.
B. Administration of antiemetics would not be indicated initially because most of such patients are in severe pain but not vomiting.
D. A serum electrolyte profile, along with other laboratory work, is indicated for diagnostic and evaluation purposes. However, this would not be the nurse’s first consideration in this particular patient population. (p. 451-452, CEN Review Manual, © 2001)

Endocrine: 5 items

1. A nurse is developing a plan of care for a patient with myxedema. The nurse would incorporate which of the following in the plan?
A. Provide a high fiber diet and encourage increased fluid intake
B. Promote a cold with lesser stimuli environment to the patient
C. Administer artificial tears as needed
D. Encourage a high-calorie, high-protein diet
Rationale: A. patient with hypothyroidism (myxedema) has a decreased gastrointestinal motility leading to constipation. Measures to prevent fecal impaction and constipation include advising the patient to drink 6-8 glasses of water every day and eating foods high in fiber such as fresh fruits, vegetables, and grains. The other choices apply to a patient with hyperthyroidism.
(MS by Black and Hawks, 7th ed. © 2005, p.1197)

2. A 36-year-old client who has undergone a subtotal thyroidectomy is subject to complications in the first 24 hours after surgery. The nurse should obtain and keep at the bedside equipment to
A. Start a cut down infusion
B. Begin total parenteral nutrition
C. Perform a tracheostomy
D. Administer tube feedings
Rationale: C. Equipment for an emergency tracheostomy should be kept in the room, in case tracheal edema and obstruction occurs. Laryngeal nerve damage can result in vocal cord spasm and respiratory obstruction. A tracheostomy set, oxygen and suction equipment, and a suture removal set (for respiratory distress from hemorrhage) made up the emergency equipment that should be readily available. (Lippincott’s Review for NCLEX-RN, 8th ed., p. 368)

3. Which of the following symptoms might indicate that a client was developing tetany after a subtotal thyroidectomy?
A. Pains in the joints of the hands and feet
B. Tingling in the fingers
C. Bleeding on the back of the dressing
D. Tension on the suture line
Rationale: B. Tetany may occur after thyroidectomy if the parathyroid glands are accidentally injured or removed during surgery. This would cause a disturbance in serum calcium levels. An early sign of tetany is numbness and tingling of the fingers or toes and in the circumoral region. Tetany may occur from 1 to 7 days postoperatively. Late signs of tetany include seizures, contraction of the glottis, and respiratory obstruction. (Lippincott’s Review for NCLEX-RN, 8th ed., p. 368)

4. The nurse asks the client to state her name as soon as she regains consciousness postoperatively after a subtotal thyroidectomy. The nurse does this to primarily monitor for signs of which possible complication?
A. Decreasing level of consciousness
B. Internal hemorrhage
C. Upper airway obstruction
D. Laryngeal nerve damage
Rationale: D. Laryngeal nerve damage is a potential complication of thyroid surgery because of the proximity of the thyroid gland to the laryngeal nerve. Asking the client to speak helps assess for signs of laryngeal nerve damage.
(Lippincott’s Review for NCLEX-RN, 8th ed., p. 368)

5. A nurse is completing an assessment on an older client who is being admitted for a diagnostic workup for primary hyperparathyroidism. Which client complaint would be the characteristic of this order?
A. Polyuria
B. Diarrhea
C. Weight gain
D. Polyphagia
Rationale: A. Hypercalcemia is the hallmark of hyperthyroidism. Elevated serum calcium level produces osmotic diuresis and thus polyuria. This diuresis leads to dehydration (weight loss rather than weight gain). (Saunders Comprehensive Review © 2006, p. 654)

Fundamentals: 10 items

1. An outpatient nurse is performing percussion on a client during a routine physical examination. Which of the following description shows a normal sound on a health client?
A. Tympanic over the LUQ
B. Resonance over the dense tissue
C. Dullness over solid mass like spleen
D. Flatness over the umbilical area
Rationale: C. Percussion sounds depend on the underlying structure being percussed. Tympany, a drum-like sound is often heard over areas with enclosed air like the bowel. Resonance is a hollow sound heard over areas of partly air and partly solid like the normal lungs. Dullness is a thud-like sound produced over a solid tissue like liver, heart, and spleen. Flatness is heard in muscles and bones which are considered dense tissue.
(Assessment made Incredibly Easy, 2nd ed. © 2002, p. 30)

2. A victim of a motor vehicle crash has sustained a severe facial injury. In assessing the intactness of cranial nerve III functioning, the nurse would
A. Observe the patient’s posturing patterns such as flexion or extension
B. Ask the client to open the mouth widely, stick out the tongue, and rapidly move the tongue side to side
C. Observe the face for symmetry and ask the patient to raise the forehead and eyebrow
D. Have the patient follow your moving finger with his eyes as you move it in different direction
Rationale: D. Having the patient follow your finger with his eyes will test extraocular movement, which is a test of cranial nerve s III, IV, and VI. (MS by Black and Hawks, 7th ed. © 2005, p. 2028)

3. You are preparing a motorcycle crash victim for a needle thoracentesis. A tension pneumothorax is suspected. After assembling the needed equipment, you would know that the insertion of a 14-gauge needle would be placed in which location?
A. Unaffected side, fourth intercostals space slightly anterior to the midaxillary line
B. Affected side, second intercostals space at the midclavicular line
C. Affected side, fifth intercostals space lightly anterior to the midaxillary line
D. Unaffected side, third intercostals space at the midclavicular line
Rationale: B. The second intercostals space at the midclavicular line is the correct location. Although a tension pneumothorax displaces the trachea and thoracic organs away from the affected side toward the unaffected side, inserting a thoracentesis needle in the unaffected side may compromise the patient. The fifth intercostals space slightly anterior to the midaxillary line would be the correct location for the insertion of a chest tube for a suspected hemothorax. This location would allow for the drainage of accumulated blood. (CEN Review Manual 3rd ed., p. 435)

4. A staff nurse asked one of the student nurse on which route of drug administration is most appropriate to use when an immediate analgesia and titration are necessary. The student nurse best response would be?
A. Intravenous (IV)
B. Sublingual
C. Patient-controlled analgesia (PCA)
D. Intramuscular (IM)
Rationale: A. the IV route is preferred as the fastest and most amendable to titration. A PCA bolus can be delivered; however, the pump will limit the dosage that can be delivered unless the parameters are changed. Sublingual and IM are also fast, but not a good route for titration. (Mosby’s Prioritization, Delegation, and Assignment ©2006, p. 16)

5. A client was prescribed with total parenteral nutrition (TPN) via central venous catheter. Which measure should the nurse take to ensure adequate assessment and to prevent infection while the patient is on TPN?
A. Administering no medications or blood products through the TPN line
B. Changing TPN tubing every 48 hours
C. Taking vital signs every 12 hours
D. Changing TPN dressing every 3 days
Rationale: A. The nurse should not administer medications, blood, or blood products through the TPN line because of possible incompatibilities between those products and any residue of the TPN solution. TPN tubing should be changed every 24 hours under strict aseptic technique. Dressing changes should be done every 48 hours. The nurse should monitor for temperature every 8 hours not 12 hours to detect spikes in temperature that may indicate infection. (Saunders Comprehensive Review © 2006, p. 141-142)

6. A client has had bone scan done. The nurse would evaluate that the client understands the elements of follow-up care if the client states that he should
A. Report any feelings of nausea and vomiting
B. Remain on bed for at least 2 hours
C. Eat a small meal for the day
D. Drink plenty of water for 1-2 days following the procedure
Rationale: D. No special restrictions are necessary after a bone scan. The client is encouraged to drink large amounts of fluid for 24-48 hours to flush the radioisotope from the systems. (Saunders Comprehensive Review © 2006, p. 999)

7. A nurse is assisting in the care of a post-op patient after a myelogram using an oil-based dye. Nursing care for the patient includes all of the following, except
A. Performing neurologic assessment frequently
B. Elevating the head 15-30 degrees for 8 hours
C. Encouraging increased fluid intake
D. Monitoring intake and output
Rationale: B. Patient who undergoes myelogram using oil-based dye must be keep flat in bed 6-8 hours. Elevating the head 15-30 degrees is indicated for water-based dye. The other nursing interventions are appropriate for the patient. (Saunders Comprehensive Review © 2006, p. 1000)

8. A nurse is reviewing the post-procedure plan of care formulated by a nursing student for a client who will undergo arthroscopy of the knee. The nurse would evaluate that the student understands the procedure when she state
A. “The client must rest entirely for the day”
B. “The client can resume exercises the following day”
C. “The client must refrain from eating food for the rest of the day”
D. “The client must be closely monitored for fever”
Rationale: D. After arthroscopy, the client usually can walk carefully on the leg once sensation has returned. The client is instructed to avoid strenuous exercises for at least a few days. The client may resume the normal diet. Signs and symptoms of infection must be reported immediately to the physician. (Saunders Comprehensive Review © 2006, p. 998)

9. A nurse is attending a client who undergoes arthrogram the previous day. Which of the following client’s complain is considered normal
A. “My joint is still puffy and painful”
B. “I feel like I’m having fever”
C. “The site is still bleeding”
D. “I observe a purulent drainage coming from the insertion site”
Rationale: A. 1-2 days after arthrogram, joint may be still edematous and tender. Instruct client to treat with icepacks and analgesics as prescribe. Notifying the physician would necessitate only, if edema and tenderness lasts longer than 2 days. (Saunders Comprehensive Review © 2006, p. 999)

10. A nurse is giving pre-op instructions to a patient scheduled for Magnetic Resonance Imaging (MRI). Which of the following statements indicate that the patient understands about the procedure?
A. “I will not eat before the procedure.”
B. “I will need to change positions during the procedure to have a full view of the scanned area.”
C. “I am not allowed to speak during the procedure.”
D. “I will be wearing earplugs during the procedure.”
Rationale: D. Patients wear earplugs during the test because of the noise created by the changes in magnetic fields. The patient is required to lie still in the procedure and a microphone inside the scanner allows the patient to speak. There is no need for the patient to be NPO the night before the procedure. (Feuer Nursing Review © 2002, p.18)

Fluid and Electrolytes: 5 items

1. A 26-year-old patient with GI bleeding requires fluid resuscitation and 4 units of packed RBC’s. Which electrolyte imbalance should the nurse monitor following the transfusion of packed RBC’s?
A. Potassium
B. Magnesium
C. Calcium
D. Phosphate
Rationale: C. The patient may suffer calcium deficiency resulting from calcium binding with citrate, which is used to prepare the packed RBC’s. Usually the patient’s calcium level is checked after the administration of multiple units of blood. (Ask a Colleague, © 2005, p. 37)

2. The nurse would anticipate the occurrence of respiratory acidosis as a result of which of the following?
A. High altitudes
B. Aldosteronism
C. Severe scoliosis
D. Salicylate toxicity
Rationale: C. Scoliosis prevents the thoracic cage from expanding and, therefore, decreases tidal volumes. Respiratory acidosis occurs in severe cases of scoliosis. High altitudes, aldosteronism, and salicylate toxicity produces respiratory alkalosis. (Springhouse: Emergency Nursing, p.155)

3. Mrs. D., age 27, is admitted to the ED after a house fire. She has second and third-degree burns over approximately 30% of her body surface area (BSA). During the initial stage of the burn injury, which fluid and electrolyte imbalances are most likely to occur?
A. Intracellular-to-intravascular fluid shift and potassium deficit
B. Interstitial-to-extracellular fluid shift and sodium deficit
C. Plasma-to-interstitial fluid shift and potassium excess
D. Interstitial-to-plasma fluid shift and sodium excess
Rationale: C. Plasma-to-interstitial fluid shift usually occurs during the initial stage of a burn injury; this causes leakage through the capillaries, resulting in edema. Because of cellular trauma, potassium is released into the extracellular space, causing hyperkalemia. (Saunders Comprehensive Review © 2006, p. 545)

4. A nurse suspects that the patient is experiencing hyperventilation. Which of the following observation will the nurse finds consistent with her diagnosis of hyperventilation?
A. Increased mental acuity
B. Tracheal deviation
C. Respiratory acidosis
D. Carpopedal spasms
Rationale: D. Patients with hyperventilation exhibit carpopedal spasms, anxiety, jaw pain, tachypnea, diffuse chest pain, confusion, diaphoresis, and headache. They exhibit respiratory alkalosis, not acidosis. (Springhouse: Emergency Nursing, p.182)

5. A patient was diagnosed of syndrome of inappropriate antidiuretic hormone (SIADH). The nurse would anticipate which of the following electrolyte imbalance?
A. Potassium
B. Sodium
C. Calcium
D. Magnesium
Rationale: B. SIADH results in hyponatremia due to fluid volume excess causing an alteration in sodium. (Saunders Comprehensive Review © 2006, p. 632)

Psych: 15 items

1. A patient is escorted to the Ed by the local police after being found in a department store preaching to the customers and stating that he is an apostle of Jesus. He is dressed in a bright fluorescent green blazer and top hat and is talking nonstop in a loud, bellowing tone, changing topics frequently. The nurse understands that the patient is exhibiting:
A. Bipolar manic disorder
B. Paranoid schizoid
C. Dissociative disorder
D. Disorganized schizophrenia
Rationale: A. This patient is exhibiting a manic phase of bipolar disease. Behaviors commonly associated with mania include elation/euphoria, agitation, and poor impulse control. They show behaviors like grandiosity, flight of ideas, provocativeness, and poor grooming. (CEN Review Manual, 3rd ed., p. 415)

2. A client with colon cancer tells the nurse. “This is ridiculous. Now they want to stage my tumor? I want to be treated immediately before this spreads. I am sick of tests. I want to be treated now.” The nurse best response to client would be
A. “Don’t worry this type of cancer spreads slowly.”
B. “You sound angry. Let’s talk about this.”
C. “Staging is needed to determine the appropriate treatment.”
D. “Ill call the physician and say that you want to begin the treatment.”
Rationale: C. Staging determines the disease progress, and is necessary to determine the type of treatment. Reflecting the client’s emotion and offering to talk may benefit but this choice is not as appropriate as the choice C. Calling the physician will not relieve the client’s anxiety. Telling the client that eh disease may progress slowly gives a false assurance and will not make him feel better.(Cracking the NCLEX-RN by J.Meyer, 7th ed. © 2003, p. 237)

3. A 40-year-old client was recently diagnosed with terminal cancer states to the nurse. “I’m really scarred…am I dying.” What would be the most appropriate response by the nurse?
A. “Why are you scared?”
B. I’m sure you’re scared. Other clients in your situation feel the same way.”
C. “You really should be careful not to let your family know you’re scared so they don’t get upset?”
D. “Tell me about what you are feeling.”
Rationale: D. This statement is a therapeutic response that encourages the client to explore his feelings. Asking “why” is non therapeutic response.

4. A client is hospitalized with obsessive-compulsive disorder. The nurse notes that the client is vigorously washing his hands. Which response by the nurse would be the most appropriate?
A. “Your hands look clean already. Why are you still washing them?”
B. “You should stop washing your hands because they will get chapped.”
C. “It’s time to go to the dining room. I’ll walk with you.”
D. “I’ll get some lotion for your hands so that they won’t get chapped.”
Rationale: C. This statement would distract the patient and would provide an alternative activity. Asking “why” question is not encouraged because client cannot explain this behavior. Choice B is threatening and implies misbehavior of the client. Choice D, does not distract the client from the behavior and leaves her in the room alone to continue the washing of hands. (Cracking NCLEX-RN by Meyer, 7th ed. p. 557)

5. A client is admitted with a diagnosis of paranoid schizophrenia. Which of the following behaviors can the nurse anticipate with the client?
A. Negative cognitive distortions
B. Impaired psychomotor development
C. Delusion of grandeur and hyperactivity
D. Alteration in apatite and sleep patterns
Rationale: D. The client with paranoid schizophrenia may feel everyone is against them, believe food is poisoned, and may remain watchful and vigilant so as not to be harmed during sleep. Client with paranoid schizophrenia do not have negative cognitive distortions. Impaired psychomotor development is seen in depressive symptoms of schizophrenia. Delusion and hyperactivity are seen frequently in manic episodes of bipolar disorder. (Cracking NCLEX-RN by Meyer, 7th ed. p. 557)

6. When communicating to a grieving family, the emergency nurse should use which of the following forms of communication?
A. Tell the family, “Everything will be all right.”
B. Use words in communicating such as “dead”, “died”, or “death”.
C. Approach the family and tell that the patient did not suffer while he is about to die.
D. Tell the family, “It was for best that the patient died rather than continue suffering”.
Rationale: B. Using words such as dead, died, or death reinforces reality, prevents denial, and supports the grief process. Telling the family that “everything will be alright” or that “it was for the best” only minimizes their feelings. Telling the family that the patient did not suffer is helpful but should be stated only if it is true. (Springhouse: Emergency Nursing, p.136)

7. A client is admitted to the hospital. During the assessment, the nurse notes that the client has not slept for a week. The client is talking rapidly and throwing his arms around randomly. What would be the highest priority in formulating a nursing care plan for this client?
A. Isolate the client until he adjusts to the hospital
B. Provide nutritious food and a quiet place to rest
C. Protect the client and others from harm
D. Create a structured environment
Rationale: B. Using the Maslow’s hierarchy of needs, the physiological needs of the client should take priority. The nurse would want to decrease stimulation, but isolation would be an extreme measure. The nurse may experience a destructive behavior but according to the Maslow’s hierarchy of needs, physiologic takes priority over safety needs. A structured environment allows the nurse to use energy in a constructive ways. However, the first priority would be to meet the physiologic, not psychosocial needs. (Cracking NCLEX-RN by Meyer, 7th ed. p. 557)

8. Which nursing diagnosis would be the priority with a client’s DSM-IV Axis 1 diagnosis of schizophrenia, paranoid type?
A. Altered protection
B. Risk for loneliness
C. Altered thought process
D. Ineffective individual coping
Rationale: C. The client is preoccupied with delusions of persecution, grandeur, ides of reference, and auditory hallucinations is predisposed to social and violent behaviors. (Cracking NCLEX-RN by Meyer, 7th ed. p. 558)

9. A client is scheduled for electroconvulsive therapy (ECT) in the morning. It is not important that the evening nurse ensures that the client does which of the following?
A. Sign an informed consent
B. Is placed on seizure precaution
C. Remember to take the morning medication
D. has a family member who will bring home any valuables
Rationale: A. Major aspects of the pre-ECT stage are obtaining lab and diagnostic data, getting an informed consent, and reinforcing client and family education. (Cracking NCLEX-RN by Meyer, 7th ed. p. 558)

10. A client is admitted with paranoid schizophrenia. Recognizing the common behaviors exhibited by a client with schizophrenia, the nurse can anticipate which of the following?
A. Grandiosity, arrogance, and distractability
B. Slumped posture and feelings of despondency
C. Disorientation, anxiety, and panic reactions
D. Withdrawal, regressed behavior, and problems with social skills
Rationale: D. Although some persons with schizophrenia are distractable, despondent, and anxious, they almost exhibit withdrawal, lack of social skills, and regressed behavior. Not all paranoid schizophrenia are distractable. Only some schizophrenia are despondent, disoriented, and anxious. (Cracking NCLEX-RN by Meyer, 7th ed. p. 558)

11. While caring for a client diagnosed with schizophrenia, the nurse know that the client may have trouble with
A. Staff who are cheerful
B. Simple direct sentences
C. Multistage command
D. Violent behaviors
Rationale: C. Client who is diagnosed with schizophrenic disorders has difficulty handling complex information, so it is best that commands are simple. The moods of the staff are insignificant. The client deals with simple direct sentences. Clients in general do not have trouble in violent behaviors. (Cracking NCLEX-RN by Meyer, 7th ed. p. 558)

12. A 28-year-old woman on a psychiatric unit is brought to the emergency department by police. He is on four-point restraints. The patient was reportedly observed running through the street naked, smashing windows and screaming. She is now calm, nonverbal, and diaphoretic and has both vertical and horizontal nystagmus. He is noted to have a 2-inch laceration to her arm. What is the priority of the nurse when assessing this patient?
A. Evaluating his mental status and obtaining a full set of vital signs
B. Performing a primary assessment and rule out other sign of trauma
C. Determining if the patient can cooperate so that restraints can be removed
D. Sending urine specimen for toxicology screening
Rationale: B. A quick primary assessment is indicated on all patients in the emergency department. This patient is a danger to himself and may have sustained other life-threatening injuries not noted by the police. The mental status exam and vital signs needs to be done but not the priority. Removing the restraints before fully examining the patient may place both the patient and emergency staff at risk. Obtaining urine specimen for toxicology screening exam takes time and should not determine the initial care. (Springhouse: Emergency Nursing, p.138)

13. When writing an assessment of a client with mood disorder, the nurse should specify
A. How flat the client’s affect is
B. How suicidal the client is
C. How grandiose the client is
D. How the client is behaving
Rationale: D. An assessment and description of a client’s specific behaviors are important in formulating an overall picture of target symptoms for intervention. Vague comments about the client’s affect are not helpful. Suicidal behavior may be documented but this is not the best response. Vague documentation of the client’s grandiosity is not helpful (Cracking NCLEX-RN by Meyer, 7th ed. p. 558)

14. Which nursing diagnosis is most likely to be associated to person with a medical diagnosis of schizophrenia, paranoid type?
A. Impaired social skills related to inadequately developed superego
B. Social isolation related to impaired ability to trust
C. Fear of being alone related to suspiciousness
D. perceptual disturbance related to delusions of persecution
Rationale: B. Client with paranoid schizophrenia frequently secludes themselves from others because of their suspiciousness, which results in their reluctance to trust people. (Cracking NCLEX-RN by Meyer, 7th ed. p. 558)

15. A client has been very despondent, withdrawn, and apathetic for about 6 months. Recently, the client began to attend outpatient clinic for treatment of depressive disorder. Fluoxetine HCL (Prozac) is prescribed, and after 3 days the client shows improvement. What is the most appropriate nursing intervention at this time?
A. Encourage the client to interact with other clients
B. Assess the client’s knowledge about the medication
C. Evaluate the potential for self-destructive behavior
D. Discuss long term plan for discharge and follow-up
Rationale: C. Antidepressant therapy may take 1-6 weeks for the client to demonstrate an improvement and reduction of suicidal behaviors. (Cracking NCLEX-RN by Meyer, 7th ed. p. 580)

Musculoskeletal: 10 items

1. A nurse visited a child with a newly dried cast. While assessing the child, the nurse understands that which of the following observation necessitates the removal of the cast before the fracture has healed?
A. If the patient can’t seem to get used of using crutches
B. If the nurse detects the formation of pressure ulcer
C. If the patient complains of itching under the cast
D. If pain is still present the day after the injury
Rationale: B. If a pressure ulcer is developing, the cast should be removed and evaluated. Itching is a common complaint and occurs as the skin dries. The patient may experience pain during the first 24 hours after the injury. Pain medications and alleviation of the injured extremity helps relive the pain. Pain that continues beyond the first 24 hours should be evaluated by the patient’s health care provider. If using crutches is a problem, the patient needs further instruction. If inability to use the crutches persists, alternate methods may be investigated, such as wheel chair. (Springhouse: Emergency Nursing, p.130)

2. A client has had a plaster of Paris cast applied to his forearm is receiving pain medication. To detect early manifestations of compartment syndrome, which of these assessment should the nurse make?
A. Observing the fingers for change in color
B. Palpating the radial pulse for pulselessness
C. Checking the cast for presence of foul odor and drainage
D. Evaluate client’s response to analgesic
Rationale: D. In compartment syndrome, secondary edema causes an increase in the pressure in the closed spaces of the tissue compartment formed by the non-elastic fascia. As the pressure increases, obstruction of the venous circulation and arterial occlusion occur, which leads to ischemia. The earliest symptom of compartment syndrome is unrelenting, progressive pain distal to the injury that is not relieved by analgesics. The pain characteristic of the individual could thus be detected by evaluating the response to analgesics. Observing the client for change in skin color could detect pallor, which is a late symptom of the syndrome. Palpating the radial pulse might allow the nurse to detect pulselessness but this is already a late sign of the syndrome. Odor and drainage in cast is more indicative of infection. (Cracking the NCLEX-RN by J.Meyer, 7th ed. © 2003, p. 182)

3. Which of the following is not an indicative of neuromuscular compromise in a patient with a shoulder dislocation?
A. Numbness of the extremity
B. Intense pain with movement of the joint
C. Inability to move the extremity
D. Paresthesia of the extremity
Rationale: B. Patients with a shoulder dislocation have pain with any movement of the joint. When there is a neurovascular compromise, the patient experiences numbness, paresthesia, and inability to move the extremity. (Springhouse: Emergency Nursing, p.119)

4. A patient presents in the ED after an assault and complains of an injury in the left femur. The nurse observes an obvious deformity to the middle of the thigh and a protruding bone. The patient’s left foot is pale and cool and palpable pulses are absent. Which of the following is the priority intervention for the patient?
A. Applying a firm in-line traction to the left leg, reassessing distal neurovascular status, and anticipating the placement of a traction splint
B. Administering oxygen at 2 liters per minute via nasal cannula
C. Attempting to push the bone back and covering with sterile dressing
D. Applying pneumatic antishock trousers to the patient and inflating the left leg compartment
Rationale: A. Applying a firm in-line traction to the left leg, reassessing the distal neurovascular status, and anticipating the placement of traction splint are the priority interventions for the patient with an open femur fracture. The use of pneumatic antishock trousers is controversial, and their value for stabilizing femur fracture has not been proved. Protruding bone should not be pushed back into the wound. Administering oxygen to a patient with a femur fracture should be done at 10-15 L/minute by way of a nonrebreather mask. (Springhouse: Emergency Nursing, p.119)

5. A 19-year-old patient is about to be delivered in the operating room for repair of a midshaft femur fracture after a vehicular accident. The nurse is aware that a possible lethal complication of femur fracture after surgery is fat embolism syndrome (FES) that usually occurs
A. 24-48 hours after injury
B. 24-72 hours after surgery
C. 24-36 hours after injury
D. 6-8 hours after injury
Rationale: B. Fat embolism syndrome (FES) is a major cause of delayed recovery and mortality after fracture. The highest incidence of FES occurs following fractures of long bones, such as femur, tibia, fibs, fibula, and pelvis. Primary manifestations typically occur within 24-72 hours after injury. (MS by Black and Hawks, 7th ed. © 2005, p. 630)

6. A 30-year-old male arrives in the ED by private vehicle after an industrial accident that resulted in a midforearm amputation. The patient has a tourniquet in place, bleeding is minimal, and the amputated part is wrapped in a towel. Which of the following is a priority intervention by the emergency nurse?
A. Preparing the patient for reimplantation
B. Caring for the severed part
C. Removing the tourniquet and controlling the bleeding part with another method
D. Obtaining an X-ray of the extremity to rule out a fracture proximal to the point of amputation.
Rationale: C. A tourniquet is an intervention of last resort to control bleeding. Removing the tourniquet is crucial to preserving as much of the stump as possible for the best therapeutic outcome. Caring for the amputated stump, reimplantation, and obtaining X-rays should follow. (Springhouse: Emergency Nursing, p.122)

7. Which of the following is an appropriate intervention for the preservation of the amputated part?
A. Discard the amputated part if it has been severed for longer than 30 minutes
B. Wrap the amputated part in a towel and moisten with cool tap water
C. Immerse the amputated part directly into a bath of ice and water
D. Anticipate the need to x-ray the severed part before reimplantation
Rationale: D. Radiologic examination should be done to rule out fractures and foreign bodies. The severed part can be reimplanted many hours after amputation. It needs to be wrapped in gauze and moistened with sterile normal saline solution. Direct immersion of the amputated part results in tissue damage secondary to thermal injury. The appropriate intervention is to clean the amputated part to remove gross debris. Then the part should be wrapped in gauze soaked with normal saline and placed in a plastic bag. The bag should be sealed tightly, placed on ice, and monitored to ensure that the tissue does not freeze. (Springhouse: Emergency Nursing, p.122)

8. A nurse is caring for a patient with newly applied cast because of sustaining a fall from the second floor of their house. Upon assessing the patient that nurse is aware compartment syndrome would likely to develop in which body part?
A. Upper arm
B. Lower arm
C. Upper leg
D. Joint spaces
Rationale: B. Compartment syndrome is most likely to occur in the lower arm, hand, lower leg, and foot. These areas have limited ability to expand with increasing tissue pressures. (Springhouse: Emergency Nursing, p.122)

9. Which of the following intervention is appropriate for a nurse to perform to control the bleeding associated with an extremity injury?
A. Applying a warm compress to cause vasoconstriction
B. Placing tourniquet 3 to 4 inches above the site of hemorrhage
C. Applying pressure dressing to the site of bleeding
D. Holding the extremity low and applying a direct pressure
Rationale: C. Applying pressure dressing to the site of bleeding is the first-line intervention for hemorrhage associated with extremity injury. Applying a tourniquet should be the last resort because it may cause tissue damage. Warm compress causes vasodilation and increases bleeding. Holding the extremity below the level of the heart also increases bleeding. (Springhouse: Emergency Nursing, p.131)

10. While caring for a client with newly applied plaster of Paris cast, the nurse makes note all of the following conditions. Which assessment finding requires an immediate notification of the physician?
A. Presence of moderate pain
B. A report of heat felt under the cast
C. Onset of paralysis in the toes of the casted foot
D. Presence of slight edema of the toes of the casted foot
Rationale: C. Unrelieved pain, excessive swelling, poor capillary refill, inability to move fingers or toes, and elevated tissue pressure indicate compartment syndrome and must be reported to the physician at once. Heat is produced by the hardening reaction of the plaster. Swelling and edema are expected responses to tissue trauma. Moderate pain related to the fracture is a common nursing diagnosis. . (Cracking the NCLEX-RN by J.Meyer, 7th ed. © 2003, p. 183)

Cardiovascular: 10 items

1. A patient with acute myocardial infarction was brought to the ED. The nurse caring for the patient is aware that the most common lethal arrhythmia in the first hour of an acute myocardial infarction (AMI) is
A. Asystole
B. Atrial fibrillation
C. Ventricular fibrillation
D. Third-degree heart block
Rationale: C. Half of the patents who die of AMI do so early, before reaching a hospital. In most of these deaths, ventricular fibrillation is the presenting rhythm. Atrial fibrillation is not considered a lethal rhythm. Asystole is a lethal rhythm, but not the most common initial rhythm associated with AMI. Third-degree heart block may be associated with MI, but is not the most common initial rhythm. (CEN Review Manual, 3rd ed., p. 4450

2. Mr. A., a 56-year-old diabetic patient with arterial insufficiency in the legs, complains that his feet are cold. Which nursing measure is contraindicated?
A. Applying a warm socks
B. Applying a heating pad to the client’s feet
C. Increasing the room temperature slightly
D. Encouraging exercise
Rationale: B. A heating pad should never be applied to the feet of a patient with compromised arterial insufficiency; tissue injury may occur before the patient feels any symptoms of it because of impaired sensations. Also, tissue demand for oxygen increases with heat; because the oxygen supply is already inadequate, applying heat may cause necrosis. The other measures are appropriate. (Saunders Comprehensive Review © 2006, p. 806)

3. A 68-year-old patient is admitted with a diagnosis of arteriosclerosis obliterans. Which statement of the patient would indicate a need for further instructions?
A. “I will start quitting from smoking”
B. “I am trying to lose weight because I’m too heavy”
C. I haven’t taken a walk for a week”
D. I will make sure to dry my feet thoroughly”
Rationale: C. The patient with arteriosclerosis obliterans should walk regularly. Walking increases exercise tolerance and may increase collateral circulation. By stating that he has stopped smoking, practices meticulous foot care, and wants to lose weight, the patient shows he understands the disease and the measures to avoid or delay complication. (Saunders Comprehensive Review © 2006, p. 806)

4. Mrs. F., age 45 is admitted with a diagnosis of thrombophlebitis of the leg. As the nurse inspects the patient, which finding should the nurse report immediately when assessing Mrs. F.?
A. Calf tenderness
B. Shortness of breath
C. Reddened skin
D. Increased skin temperature of the affected leg
Rationale: B. Shortness of breath is the initial presenting symptom of pulmonary embolism, a serious and potentially life-threatening complication of deep vein thrombosis. Increased skin temperature of the affected leg, calf tenderness, and reddened skin are expected findings of thrombophlebitis that do not require immediate action. (Saunders Comprehensive Reviewer © 2006, p.804)

5. A 67-year-old client was rushed to the hospital and was later diagnosed of congestive heart failure (CHF). The patient is in the acute phase of ventricular heart failure. To alleviate his symptoms, the nurse should place him in
A. Left lateral Sim’s position
B. Dorsal recumbent position with elevated feet
C. Low Fowler’s position with elevated knees
D. An upright position
Rationale: D. In left ventricular failure, the left ventricle cannot pump the necessary volume of oxygenated blood coming from the lungs, resulting in lung congestion. An upright position allows full chest expansion, which helps relieve dyspnea. The dorsal recumbent position decreases ventilation; elevating the feet increases blood flow to the heart, putting a greater workload on it. The low Fowler’s position with elevated knees may cause pooling of blood in the abdominal area, which may lead to increased ascites and poor diaphragmatic contractions. The left lateral Sim’s position has not been proven more effective in emptying the right side of the heart; besides an increased in the amount of blood pumped from the right ventricle into the pulmonary circulation would only worsen the patient’s condition. (Saunders Comprehensive Review © 2006, p. 797)

6. Mr. S. a 48-year-old banker, is admitted to the emergency department (ED) with severe chest and arm pain of 4 hours duration that radiates to his jaw, accompanied by sweating and weakness. While assessing Mr.S. the nurse should first relieve his discomfort by:
A. Checking his arterial blood gas
B. Administering 4 mg of morphine IV
C. Instructing him in guided imagery
D. Starting oxygen by nasal cannula at 2 liters/min
Rationale: D. By starting oxygen, the nurse improves the patient’s arterial oxygen saturation, supplying the myocardium with needed oxygen to minimize tissue damage caused by ischemia. Most emergency departments can allow the nurse to begin oxygen without the physicians order, and 2 liters/min is a safe flow rate even for patient with pulmonary disease. Although, the patient may need morphine, the nurse cannot administer it ―or start any IV lines―without a physician’s order. Checking the arterial blood gas may help the physician in diagnosing the patient’s problem but will not help to relieve discomfort. Guided imagery can be beneficial in relieving anxiety and pain, but the nurse must relieve the patient’s initial pain and establish a trusting relationship with him before the imagery is useful. (Saunders Comprehensive Reviewer ©2006, p. 795)

7. Mr.S. exhibits signs and symptoms of cardiogenic shock during the cardiac catheterization procedure. The nurse is aware that characteristics of cardiogenic shock includes all of the following, except
A. ECG showing ST-segment elevations in leads II, III, and aVF
B. Pain in the distal, left lower extremity
C. Disorientation to time and place
D. Narrowed pulse pressure and thready pulse
Rationale: B. Pain in the distal, left lower extremity more likely indicates embolism; it is not characteristic of cardiogenic shock. Disorientation, narrow pulse pressure and thready pulse, and ST segment elevations in lead II, III, and aVF are manifestations of cardiogenic shock. (MS by Black and Hawks, ©2005, p. 1715-1716)

8. A 50-year-old patient comes to the hospital complaining of severe “tearing” chest pain that radiates to the back. The nurse would probably suspect that the patient is experiencing
A. Acute myocardial infarction
B. Pericarditis
C. Dissecting abdominal aneurysm
D. Ruptured appendicitis
Rationale: C. Sudden onset of severe chest pain that radiates to the back with a tearing sensation is a classic sign of abdominal aneurysm. (Springhouse: Emergency Nursing, p. 24)

9. A male arrives in the emergency department after a scuba diving trip with friends. He is complaining of shortness of breath, chest pain, and vertigo. He is coughing up pink, frothy sputum. The emergency nurse should assess the patient while placing him in what position?
A. Right lateral decubitus
B. High Fowler’s position
C. Semi Fowler’s position
D. Left lateral trendelenburg
Rationale: D. The patient is experiencing an air embolism caused by failure to exhale on ascent. He should be placed in trendelenburg in the left lateral decubitus position to avoid cerebral embolus. (Springhouse: Emergency Nursing, p. 42)

10. A trauma patient with a diagnosis of ruptured bladder has two large bore IV lines, oxygen, a nasogastric tube, and an indwelling urinary catheter in place. Initial ital signs were blood pressure 120/54 mmHg, pulse 120 beats/minute, respirations 20 breaths/minute, pulse oximetry 95% (room air), and temperature 98.20 F. Which of the following might indicate an impending hypovolemic shock?
A. Pulse of 80 beats/minute
B. Pain that is unrelieved
C. Restlessness
D. Blood pressure of 100/80 mmHg
Rationale: C. Restlessness is often the first sign of impending hypovolemic shock or hypoxia. A pulse rate of 80 brats/ minute is still considered normal. Pain is normal to a patient with trauma. The blood pressure is within normal limit. (Springhouse: Emergency Nursing, p. 79)

Respiratory: 5 items

1. A 19-year-old college student who sustains a fractured ribs on the right side of his chest after a football game is taken to the emergency department. To promote pulmonary ventilation, the nurse plans to include all of the following, except
A. Encouraging coughing and deep breathing
B. Providing hydration
C. Positioning the client on his right side
D. Encouraging the use of an incentive spirometer
Rationale: C. Positioning the client on his right side is inappropriate because is fractured ribs are on this side; this position would increase the patient’s pain, interfering with chest expansion. Coughing, deep breathing, and use of incentive spirometer help inflate the lungs and remove secretions, thereby improving pulmonary ventilation. Hydration makes secretions less viscous and tenacious, which enhances their removal and improves ventilation. (Nurse Test: med-Surg.p.21)

2. Mr. D., a client with chronic emphysema is admitted in the unit with acute respiratory failure. Endotracheal intubation and mechanical ventilation with high peak respiratory pressures are attached. After several days in the hospital, which physiologic parameter indicates that weaning Mr. D., from the ventilator is already feasible?
A. An increase in shunt function
B. Vital capacity of less than 10 ml/kg of body weight
C. Negative peak inspiratory pressure greater than―20 cmH2O
D. PaO2 of 100 mmHg on 100% oxygen
Rationale: C. The peak inspiratory pressure is measured by attaching an aneroid manometer to the endotracheal or tracheostomy tube and then having the patient, after a maximal exhalation, inhale as forcefully as possible. Negative pressure greater than ―20 cmH2O indicates adequate inspiratory muscle strength. The vital capacity should be measured easily with a respirometer. The vital capacity should be greater than 10 ml/kg of body weight to avoid atelectasis. Arterial oxygenation (PaO2) on 100% oxygen would be close to 600 mmHg in a person with normal respiratory functioning. An increase in shunt fraction indicates worsening condition, which would preclude weaning. (Nurse Test: Med-Surg. P.26)

3. A 29-year-old patient with multiple trauma and hemothorax arrives in the emergency department. A chest tube is placed. Initially 500 mL of blood drains from the tube. The patient’s vital signs are blood pressure 140/70 mmHg and a heart rate of 138 beats/minutes. Respirations are controlled by mechanical ventilation at a rate of 16 breaths/minute. Which assessment parameter should be closely monitored over the next hour?
A. Central venous pressure
B. Vital signs
C. Chest tube drainage
D. Urine output
Rationale: C. The initial drainage from the chest tube is 500 mL, so the chest tube drainage should be monitored closely. If the patient continues to lose blood at a rate of 200 mL/hour or more, the patient may require surgical intervention to identify and repair the source of bleeding. Monitoring vital signs, central venous catheter, and urine output are all important in assessing the fluid status, but monitoring the chest tube drainage is the priority. (Springhouse: Emergency Nursing, p.165)

4. An elderly with chronic obstructive pulmonary disease (COPD) is given discharge instructions regarding nutritional support. Which of the following statements identifies the need for further teaching?
A. “I should rest for 30 minutes before each meal.”
B. “I will limit my fluid intake at meal time.”
C. “I should eat five or six small meals each day.”
D. “I should select most of my foods from the carbohydrate group.”
Rationale: D. the patient with COPD should rest 30 minutes before each meal to conserve energy and decrease dyspnea. The patient should also avoid exercise and breathing treatments for at least 1 hour before or after eating. The patient with emphysema has a markedly increased need for protein and calories to maintain an adequate nutritional status, and should be divided into five or six small meals a day, and fluid intake should be maintained 3 L/day unless contraindicated. Fluids should be taken between meals to reduce gastric distention and pressure on the diaphragm. (Springhouse: Emergency Nursing, p.170)

5. The goals of successful asthma management include the measurement of lung function. This can be accomplished with peak expiratory flow rate (PEFR). The nurse understands that the optimal PERF is
A. PERF variability of less than 30 %
B. PERF less than 50% of predicted or personal best
C. PERF variability of 80% to 90%
D. PERF greater than 80% of predicted or personal best
Rationale: D. The optimal PERF is greater than 80% of predicted or personal best with a variability of less than 20%. Monitoring PERF helps assess the severity of obstruction. The nurse should evaluate the patient’s response and detect changes in airflow. If PERF is increasing and subjective symptoms are decreasing, there is no need to change the medication or dosage. (Springhouse: Emergency Nursing, p.173)

Infectious/ Precaution: 15 items

1. Mr. E., age 29, is referred to the local hospital clinic after a positive tuberculin test result. The TB diagnosis is confirmed, and Mr.E., is put on respiratory isolation. Mr.E, asks why this is necessary. The nurse should tell the patient that
A. TB is transmitted via inhalation of droplets containing diseased nuclei
B. TB is like other respiratory infections
C. Close contracts is necessary for TB transmission
D. persons who has been exposed to TB are sensitized
Rationale: A. Speaking, coughing, or sneezing expels small droplets of moisture, which have small particles of respiratory secretions as nuclei. Inhalation of air with the nuclei can cause infection of certain diseases, such as TB. Unlike other respiratory infection, TB does not require close contact (with in 3 feet) or immediate exposure to the airborne nuclei for the disease to be transmitted; therefore, isolation techniques are necessary. People who are exposed developed antibodies, but this does not necessarily mean they will not contract the disease. (Saunders Comprehensive Review © 2006, p. 740)

2. When providing care for a TB patient under isolation period, the nurse must
A. Instruct the patient to wear mask
B. Wear gloves in performing procedures
C. Wear a special individually fitted mask
D. Wash hands and dispose isolation gowns properly
Rationale: C. By wearing a face mask that is individually fitted covers the nose and mouth prevents the nurse from inhaling the tubercle bacilli. Rubber gloves and isolation gowns are indicated when direct contact with secretions containing the TB organism is possible. Frequent hand washing is encourage before and after caring the patient. (Saunders Comprehensive Review © 2006, p. 740)

3. Which of the following is necessary for nurses to protect themselves from HIV contamination?
A. Respiratory precautions
B. Enteric precautions
C. Frequent hand washing
D. Universal precautions
Rationale: D. Universal precautions are a requirement of care to protect both patients and nurse of HIV. . (Cracking the NCLEX-RN by J.Meyer, 7th ed. © 2003, p. 248)

4. When teaching an HIV-positive person about HIV transmission prevention, which of the following should the nurse include?
A. The HIV-positive person can share the same bathroom with the other household members
B. The HIV-positive person can share razors and toothbrushes with the other household members
C. The HIV-positive laundry must be done separately
D. The HIV-patient should have his or her own utensils
Rationale: A. Persons sharing households with HIV-positive persons are at very low risk to become infected if good hygiene is performed. Any articles that may come in contact with blood should not be shared. Washing dishes and clothes in hot, soapy water kills HIV. . (Cracking the NCLEX-RN by J.Meyer, 7th ed. © 2003, p. 246)

5. An AIDS client with open Kaposi’s sarcoma lesions on the upper extremities is admitted to the hospital. When assessing the client’s vital signs, the nurse should wear which of the following?
A. Gloves and goggles
B. gloves and gown
C. Gloves only
D. Gown only
Rationale: C. Gloves are the only barrier needed. . (Cracking the NCLEX-RN by J.Meyer, 7th ed. © 2003, p. 249)

6. An end-stage AIDS client requires suctioning. When performing this task, the nurses is correct to wear
A. A sterile gloves and eye protection
B. A mask and eye protection
C. A mask, eye protection, and sterile gloves
D. A mask and sterile gloves
Rationale: C. When suctioning a client, a mask, sterile gloves, and eye protection must be worn in order to protect the nurse’s mucus membranes of the eyes, mouth, and nose. . (Cracking the NCLEX-RN by J.Meyer, 7th ed. © 2003, p. 249)

7. Which of the following nursing diagnoses would be most appropriate when planning interventions for a 27-year-old female with a CD4 cell count of 800 mm3 and facial Kaposi’s sarcoma?
A. Risk for infection related to low immune system
B. Body image disturbance related to facial Kaposi’s sarcoma lesions
C. Potential for impaired mobility related to lower extremity Kaposi’s sarcoma lesions
D. Potential for sexual dysfunction related to genital Kaposi’s sarcoma lesions
Rationale: B. Kaposi’s sarcoma can occur anytime during the course of HIV because it is not directly linked to immunosuppression. It can occur in clients who are relatively well. Body image disturbances are significant issues for clients with dermal Kaposi’s sarcoma. Normal value for CD4 cell count is 800 mm3 and opportunistic infection may contract the patient irregardless of CD4 count. . (Cracking the NCLEX-RN by J.Meyer, 7th ed. © 2003, p. 248)

8. A 6-year-old child has chicken pox and the mother is asking when the child can return to school. The nurse should respond
A. “In 7 days, after the lesions have all crusted.”
B. “In 14 days, once the virus has passed the incubation period.”
C. “After the antibiotics have been given for 2 full days.”
D. “In 3 days, after new vesicles have stopped appearing.”
Rationale: A. Children may return to school once the lesions have crusted over, generally 7-10 days after the onset of rash. . (Cracking the NCLEX-RN by J.Meyer, 7th ed. © 2003, p. 351)

9. An 8-year-old child has head lice. The nurse should instruct the parents to do which of the following?
A. Keep the child in home from school for 3 days
B. Discard combs and brushes
C. Get the child a haircut
D. Placed all stuffed toys in a black plastic bag for 24 hours
Rationale: D. All stuffed toys should be placed in a black plastic bag for 24 hours so that the nits that could be on them are killed. Children may return to school the first day after initial treatment. Combs and brushes should be soaked in a pediculocidal agent for 15 minutes. Hair cuts are not required. . (Cracking the NCLEX-RN by J.Meyer, 7th ed. © 2003, p. 351)

10. Which of the following statements would be helpful in teaching a family about caring for an HIV-positive infant at home?
A. The virus cannot be transmitted by casual contact
B. Everyone in the house should receive a prophylactic medication
C. Sterilize all toys and utensils after the chills has used them
D. Wash the child’s clothes separately from the rest of the family
Rationale: A. the virus cannot be transmitted through casual contact. This must be reinforced so that the child is not deprived of touch. . (Cracking the NCLEX-RN by J.Meyer, 7th ed. © 2003, p. 399)

11. A 25 year-old symptomatic client in the ED shows a positive Clostridium defficile infection. The nurse would properly institute what type of precautionary measure to the client?
A. Blood and body fluid precautions
B. Contact precautions
C. Standard precautions
D. Airborne precautions
Rationale: B. Clostridium defficile is easily transmitted and commonly results from changes in the GI tract’s normal flora, a consequence of antibiotic therapy. It is transmitted person-to-person or hand-to-mouth from contaminated object. Minimizing or preventing antimicrobial exposure and restricting antibiotics known to cause diarrhea (clindamycin) is one approach that can reduce the patient’s risk for developing it. Placing symptomatic patient on Contact precautions, using single-patient-only disposable thermometers (especially rectal thermometers) for symptomatic patients, strict hand washing with an antimicrobial soap and water (alcohol-based hand sanitizers are not effective against spore forming C. defficile bacteria), and effective cleaning of symptomatic patients are other approaches to prevent the transmission of C. defficile. (Ask a Colleague © 2005, p. 28)

12. A nurse is taking care of an infant with suspected Rotavirus infection. The nurse is aware that the appropriate precaution to be implemented is
A. Droplet precaution
B. Enteric precaution
C. Contact precaution
D. Blood and body fluid precaution
Rationale: B. Rotavirus commonly causes acute infantile gastroenteritis leading to diarrhea in young children. (The Lippincott Manual of Nursing Practice © 2001, p. 1466)

13. A nurse is giving discharge teaching for a patient with hepatitis C. Which of the following patient’s response indicate a need further teaching?
A. “The disease can be acquired through renal dialysis.”
B. “IV drug users are prone to the infection.”
C. “I should avoid in engaging multiple sexual partners”
D. “The disease can be transmitted through breast feeding.”
Rationale D. Hepatitis C is formerly called non-A, non-B hepatitis. Mode of transmission in most cases is through blood or blood products, usually after commercial or paid blood donors. It is the most common form of posttransfusion hepatitis. Risk factors include IV drug users, renal dialysis, sexual intercourse, contaminated piercing and tattooing. Transmission through breast milk occurs in hepatitis B infection. (The Lippincott Manual of Nursing Practice © 2001, p. 683)

14. A mother asks the nurse about appropriate care and intervention for her10-year-old child with scabies. Which of the following responses made by the nurse is inappropriate?
A. “Observe drainage/secretion precautions for the first 24 hours of therapy.”
B. “The disease may predispose the child to acute glomerulonephritis.”
C. “Bed linens, towels, and clothing used by the child should be placed in a black plastic bag and properly discarded.”
D. “Crusts and debris can be removed from the affected area using tap water wet compress.”
Rationale: C. Regular laundering of bed linens, towels, and clothing is enough to prevent the transmission of infection. (The Lippincott Manual of Nursing Practice © 2001, p. 1603)

15. A 9 year-old child with suspected measles (rubeola) is being admitted to the hospital. In preparing for the admission of the child, you plan to implement which precautionary measures for the child?
A. Placing the child in a private room, wearing gloves whenever you enter the room, limiting patient transport and movement out of the room.
B. Implementing strict hand washing, washing utensils thoroughly, and stool and needle precautions.
C. Placing the child in a private room, instructing visitors to wear masks, proper disposal of contaminated articles
D. Instructing visitors to wear gowns and masks when entering the room, frequent hand washing, proper bagged and labeling of contaminated articles.

Rationale: C. Rubeola is transmitted via airborne particles or direct contact with infectious droplets. Respiratory precautions are required, and those in contact with the child should wear masks. Gowns and gloves are not indicated. Articles that are contaminated should be bagged and labeled properly. Enteric precautions are not needed. (Saunders Comprehensive © 2006, p. 515)

Neurologic: 10 items

1. Mr. C., a 68-year-old retired assembly-line worker, is admitted to the hospital with a diagnosis of Parkinson’s disease. Which clinical features of the disease should the nurse expect to observe during admission assessment?
A. Dry skin and drooling
B. Muscle flaccidity and lethargy
C. Swayback stance and excessive perspiration
D. Stiffness and muscle cramps
Rationale: D. Muscular rigidity (stiffness) and restlessness, not muscular flaccidity and lethargy, are characteristics of Parkinson’s disease. Drooling may result from decreased frequency of swallowing because of facial and pharyngeal muscle rigidity; however, the skin is not usually dry but oily, probably from hypothalamic dysfunction. Also, many patients perspire excessively because of the disorder of the hypothalamic heat-regulator mechanism as well as impairment of the perspiration control. Muscle cramps in the legs, neck, and trunk are common, probably because of muscle rigidity, however, difficulty in maintaining balance results in a stoop-over position, not a swayback stance. (MS by Black and Hawk © 2005, p. 2172)

2. Mr. A., a 65-year-old tailor, is admitted to the hospital, complaining of difficulty holding objects. He states that his condition has progressively worsened. The physician suspects amyotrophic lateral sclerosis (ALS). In order to confirm the diagnosis, the physician would order which of the following?
A. Rombergs test
B. Lumbar Puncture
C. Creatine phospokinase (CPK) test
D. Skeletal muscle biopsy
Rationale: D. A skeletal muscle biopsy identifies the presence or absence of the degenerative muscle fibers and differentiates between neurogenic and amyotrophic disease. Serum enzymes, such as creatine phospokinase (CPK), are present in all muscle tissue; an increased level indicates damage but does not confirm the diagnosis. Lumbar puncture in patient with ALS is normal. The Romberg test identifies the position sense after a cerebral concussion; it does not confirm ALS. (NSNA by Stein, 5th ed., p. 228)

3. Characteristic clinical manifestations of ALS include which of the following, except?
A. Fasciculation of the involved muscle
B. Loss of bowel and urine control
C. Uncontrolled outburst of crying
D. Dysphagia and dysarthria
Rationale: B. Patients with ALS usually does not lose urinary and bowel sphincter control because the disease does not affect the spinal nerves controlling these muscles. However, other muscles that control the neck, pharynx, larynx, trunk, and legs atrophy from the gradual degeneration of motor neurons; this can lead to dysphagia, dysarthria, uncontrolled outburst of crying or laughing, and fasciculations of involved muscles. (MS by Black and Hawks © 2005, p. 2185)

4. After confirming the diagnosis of ALS, the nurse caring for Mr. A., prepares for discharge teaching. The patient-teaching session should cover all of the following topics, except:
A. Wearing well-fitting, supportive shoes
B. Avoiding fatigue and extreme cold
C. Encourage an increased fluid intake
D. Careful monitoring for presence of pressure ulcers
Rationale: D. Pressure ulcers are not usually a problem because the sensory system remains intact and the patient can still feel when the pressure on the body part is too great. Because muscle atrophy leads to poor peristalsis, the patient is prone to constipation; a high residue diet and frequent fluid intake will help correct this. The patient should avoid extreme cold or hot which may cause transient muscle weakness; he should also pace his activities to prevent fatigue. Well-fitting, supportive shoes with the aid of cane or walker in ambulation will help the patient maintain his balance and prolong independence. (MS by Black and Hawks © 2005, p. 2185)

5. Ms. F., a 23-year-old secretary is admitted to the hospital because of extreme muscle fatigue on exertion. Her physician suspects myasthenia gravis. Which assessment findings are most likely seen in a patient with myasthenia gravis?
A. Restlessness, decreased level of consciousness, and a history of extreme muscle weakness in the morning
B. Unequal pupillary response, diplopia, and inability to hold her mouth closed
C. Frequent changes in facial expression, exopthalmos, and a low-pitched voice
D. Fatigue, dysphagia, and a soft, nasal voice
Rationale: D. Myasthenia gravis (MG) affects the muscles used in chewing, causing dysphagia. This also destroys acetylcholine sites in muscles used in speaking; this weakens muscle contractions, resulting in a soft, nasal voice. The patient may be restless because of respiratory difficulty and anxiety; however, muscle weakness usually worsens in the afternoon and evening, and a decreased in the level of consciousness s not common. MG may cause diplopia or the inability to hold the mouth closed if it affects the lateral rectus, masseter, medial pterygoid, and temporalis muscles; however pupillary response remains normal because the disease does not affect the ciliary muscle. Exopthalmos and low-pitched voice are not common with MG; weakness of the facial muscles typically prevents changes in facial expression. . (MS by Black and Hawks © 2005, p. 2182)

6. While providing morning care, the nurse observes that Ms.F. is wheezing and pale with a respiratory rate of 40 breaths/minute. The nurse immediately:
A. Administer oxygen at 2 liters/ minute via nasal cannula
B. Notify the physician immediately
C. Have the patient cough and deep breathe every 2 hours
D. Administer 500 mg of aminophylline IV
Rationale: B. Because ineffective breathing pattern and airway clearance are life-threatening complications of myasthenia gravis, the nurse should notify the physician immediately. The physician would probably order arterial blood gas analysis after being notified; the nurse should not administer oxygen before hand to ensure the accuracy of the result. Aminophylline, which relaxes the smooth muscles of the bronchial airways, must be ordered by the physician; it is given only after the patient’s status is evaluated and myasthenic and cholinergic crises have been ruled out. . (MS by Black and Hawks © 2005, p. 2183)

7. Mrs. J., a 72-year-old widow with Alzheimer’s disease, is brought to the emergency department by her daughter after falling at an adult care center. In planning nursing intervention for Mrs. J., what is most important to be considered?
A. Reinforcing timely personal self-care habits
B. Halting the deterioration process
C. Keeping her comfortable
D. Providing a safe environment
Rationale: D. Alzheimer’s disease causes physical and mental disabilities, placing Mrs.J. at high risk for accidents and infection; the nurse must prioritize the provision of a safe environment to prevent accident. Good personal hygiene is desirable but not as important as a safe environment. (Saunders Comprehensive Review © 2006, p. 1089)

8. Mrs. K., a 45-year-old schoolteacher is admitted to the unit with symmetrical weakness of her lower extremities. After resting, the physician diagnoses Guillain-Barré syndrome. Routine assessment for Mrs.K., should include checking for:
A. Deterioration in hearing
B. Ptosis and facial weakness
C. Impaired gag and swallowing reflex
D. Facial twitching and tremors
Rationale: C. Guillain-Barré syndrome may lead to loss of gag and swallowing reflexes, which greatly increases the risk of aspiration. The other choices are not associated with this disorder. The patient with Guillain-Barré syndrome typically complains of hypercusis (abnormal sensitivity to sound), not impaired hearing.
(MS by Black and Hawks © 2005, p. 2182)

9. A teenager is found lying supine in the sidewalk and is brought to the emergency department for evaluation. The patient smells ethanol and complains of head and neck pain and dos not recall events leading to his arrival in the emergency department. On arrival the patient is tested and has a Glasgow Coma scale of 14. A hematoma is palpated to the occipital and frontal skull areas. Vital signs show blood pressure of 160/88 mmHg, pulse 108 beats/minute, respirations 18 breaths per minute and regular, and temperature 98.80F. What is the priority intervention for this patient?
A. Perform a head-to-toe assessment
B. Apply a cervical immobilization
C. Obtain a specimen to determine the blood alcohol level
D. Administer analgesics for pain
Rationale: B. Immobilization of the head and neck reduces the risk of further damage to the cervical spine. All patients with suspected head and neck trauma should be immobilized until all seven cervical vertebrae are cleared by X-ray visualization. A complete head-to-toe assessment (secondary survey) should be performed when airway, breathing, and circulation are assessed. Administering analgesics in a patient with altered mental status or head injuries is not a priority intervention. Determining the level of alcohol in the blood is a useful information but not priority. (Springhouse: Emergency Nursing, p. 83)

10. A man who fell from a 20-feet building is determined to have a spinal cord fracture with spinal cord transaction at the level of C-6. This injury results in which of the following findings?
A. Quadriplegia with loss of respiratory function
B. Quadriplegia with diaphragmatic breathing and gross arm movements
C. Bowel and bladder dysfunction
D. Paraplegia with variable loss of intercostals and abdominal muscles
Rationale: B. A patient with an injury at the level of C-6 has quadriplegia with diaphragmatic breathing and gross arm movements. The patient may also suffer from hypotension and an atonic bladder. (Springhouse: Emergency Nursing, p. 89)

MCN/G&D: 11 items

1. A 4-year-old boy is scheduled for an angiogram. What can the nurse tell the child to better prepare him for the procedure?
A. Telling the child ahead that he will go for the test in 2 hours
B. Explaining that the doctor will look at his heart through a hole in his leg
C. Explain that the room will be cold and that the table will feel hard
D. Promise him that he will be fine through out the procedure
Rationale: C. Piaget described the preoperational thinking of a 2-7 year old as concrete and tangible. At a 4-year-olds developmental level, time is still incompletely understood; the child interprets it according to his own frame of reference. . (Cracking the NCLEX-RN by J.Meyer, 7th ed. © 2003, p. 341)

2. When counseling parents about the nutritional habits of toddlers, the nurse should include which of the following?
A. If toddlers that they don’t want a food, it should be substituted for one that they request
B. Between meal snacks should be avoided
C. Toddlers should be given the same utensils and dishes everyday
D. Foods should be designated as a reward
Rationale: C. Toddler’s needs for routine/ritualism should be incorporated into feeding practices. Substitution should be made sparingly in order to avoid catering to the changing whims of toddlers. Toddlers enjoy grazing and the provision of nutritious snacks can help to maximize nutrition. The use of food as a reward encourages overeating. . (Cracking the NCLEX-RN by J.Meyer, 7th ed. © 2003, p. 342)

3. A nurse evaluates the following babies in the clinic. Which baby may be experiencing a developmental delay?
A. A 10-month-old who does not sit
B. An 18-month-old who does not walk
C. A 3-month-old who does not crawl
D. A 1-month-old who does not coo
Rationale: A. An infant should be able to sit unsupported by 7 or 9 months. Therefore, further assessments should be made of the 10-month-old. Cooing begins around 2 months of age. Crawling begins around 7 or 8 months of age. At about 9 months, infants begin to cruise and walk around furniture. . (Cracking the NCLEX-RN by J.Meyer, 7th ed. © 2003, p. 418)

4. Which of the following toys is most appropriate for a 12-moth-old infant?
A. Tricycle
B. Paints and crayons
C. Push/pull toy
D. Musical mobile
Rationale: C. Toys that can be pushed or pulled along are most appropriate since they aid in walking and are enjoyed by children learning to walk. Tricycles are appropriate for 3-4 year-old children. Paints and crayons are suited for toddlers and preschoolers. Musical mobiles are best for young infants. . (Cracking the NCLEX-RN by J.Meyer, 7th ed. © 2003, p. 419)

5. A nurse provides home care instructions to the parents of a child with celiac disease. The nurse teaches the parents to include which of the following food items in the diet plan?
A. Egg sandwich and orange juice
B. Cornflakes with milk
C. Crackers and milk
D. Cookies and ale
Rationale: B. Foods allowed for celiac disease are those considered gluten-free such as beef, pork, poultry and dairy products, fish, egg, fruits, vegetables, rice, and corn. All wheat, rye, barley, and oats with their by product should be eliminated. (Saunders Comprehensive Review © 2006, p. 452)

6. In inserting a nasogastric tube (NGT) in a 3-month-old infant, which of the following will the nurse considers? (Choose all that apply)
A. Stabilizing the tube by taping it in the infants cheek
B. Measuring the length of the tube from the nose to the bottom of the earlobe and to the end of xyphoid process
C. Assessing the tube placement by aspirating color and pH of gastric fluids
D. Insertion through the mouth is more preferable
E. The use of microwave in warming t formula is advised for infants
F. Administering the formula at the flow rate of 10 ml/minute
Rationale: ABCDF. All of the following are appropriate measures in inserting NGT to infants except warming the formula using a microwave. (Wong’s Essential’s of Pediatric Nursing, 6th ed.,p. 812-813)

7. A nurse in the maternity unit taking care for a laboring woman for 6 hours identifies that the woman is already on her second stage of labor when she observes which o the following?
A. Presence of an increased bloody show
B. The client begins to expel clear vaginal fluid
C. Uterine contractions that lasts for 60 seconds
D. Cervical dilatation of 8 cm
Rationale: A. The second stage of labor begins when the cervix is fully dilated and ends with the birth of the neonate. Uterine contractions usually last for 60-75 seconds. An increased bloody show occurs and the mother feels an urge to bear down. (Saunders Comprehensive Review © 2006, 300)

8. A nurse is preparing the admission of a woman in the3rd trimester of pregnancy, who is experiencing vaginal bleeding and has a suspected diagnosis of placenta previa. The nurse reviews the physicians order and would question which of the following?
A. Attaching the patient in an external electronic fetal heart rate monitor
B. Prepare to draw a hemoglobin and hematocrit
C. Preparing the client for an ultrasound
D. Preparing equipment for manual pelvic examination
Rationale: D. Performing a manual pelvic examination is contraindicated when vaginal bleeding is apparent in the third trimester until a diagnosis is made and placenta previa is ruled out. (Saunders Comprehensive Review © 2006, p.317)

9. A nursing instructor asks a nursing student about the appropriate play for a 5-year-old child. The student nurse shows understanding of the appropriate growth and development when she state
A. Monopoly
B. Simple handicraft set
C. Musical mobiles
D. Chess board
Rationale: B. Manipulative, constructive, creative, and educational toys provide for quiet activities, fine motor development, and self-expression in pre-schoolers. Easy construction sets, large blocks of various sizes and shape, counting frame, alphabet or counting number flash cards, simple carpentry tool, simple sewing or handicraft sets are suitable for pre-schoolers. Monopoly and chess board are appropriate for school age child. and musical mobiles are appropriate for infants. (Wong’s Essentials of Pediatric Nursing, p.443)

10. The mother of an infant reports to the nurse that her baby is having diarrhea and flatulence 30 minutes after feeding. The nurse suspects that the infant may be experiencing lactose intolerance and recommend an alternative feeding like
A. Breast milk
B. Soy-based formula
C. Powdered cow’s milk formula
D. Low fat milk
Rationale: B. lactose intolerance refers to the deficiency of the enzyme lactase which is needed for the hydrolysis or digestion of lactose in the small intestine. Treatment of lactose intolerance is elimination of offending dairy products by substituting soy-based formula to cow’s milk or human milk.
(Wong’s Essentials of Pediatric Nursing, p.399)

11. A 2-year-old child is brought to the ED after being found submerged in the neighborhood pool. The child is unconscious with a pulse of 60 beats/ minute and respirations of 4 breaths/ minute. Breathing is being assisted with a bag-valve-mask device. What should be the first action of the emergency nurse?
A. Assist with intubation
B. Administer epinephrine
C. Insert IV access to infuse 500 ml of D5W
D. Obtain a cervical spine film
Rationale: A. Airway, breathing, and circulation are always first priority actions. Therefore, the emergency nurse should first assess with the intubation noting that the child’s respiration is declining. Once respirations have been stabilized, I.V. access can be attempted. Epinephrine is the drug of choice in a child with refractory shock. An infusion of 1 mg in 500 ml D5W should be started and titrated to effect. Once the patient is stabilized, a cervical spine film can be obtained. (Springhouse: Emergency Nursing, p. 42)

Pharmacology: 20 items

1. To treat for a patient’s intravascular volume deficit, the physician orders 1,000 ml of lactated Ringer’s solution to be infused at 70 gtt/minute. The drop factor is 10 gtt=1 ml. how many milliliters per hour will be infused?
A. 721 ml/hour
B. 500 ml/hour
C. 420 ml/hour
D. 388 ml/hour
Rationale: C.
= 420 ml/hr

Step 1: Change drops to milliliters
10 gtts: 1 ml:: 70 gtts: x ml
10 x= 70
x=
x= 7 ml

Step 2: Calculate the number of milliliters per hour.
1 min: 7 ml:: 60 min: x ml
x= 7 x 60
x= 420 ml/hr
(Nurse Test: Dosage Calculation, p. 22)

2. The nurse follows a physician’s order to administer 30mg/minute of 50 ml of dextrose 5% in ½ normal saline solution, to which 1 g of azlocillin (Azlin) has been added. What will be the flow rate of this extended-spectrum penicillin if the drop factor is 1 ml= 60 mcgtt?
A. 60 mcggt/minute
B. 90 mcggt/minute
C. 110 mcgtt/minute
D. 140 mcgtt/minute
Rationale: B
Remember: 1 g= 1,000 mg
= = 90 mcgtt/minute

Step 1: Change milliliters to microdrops.
1 ml: 60 mcgtt::50 ml: x mcgtt
X= 60 50
X= 3,000mcgtt

Step 2: Calculate required minutes.
30 mg: 1 min:: 1,000 mg: x min
30x=1,000
X=
X= 33.3 minutes

Step 3: Calculate the flow rate.
33.3 min: 3,000 mcgtt:: 1 min: x mcgtt
33.3x= 3,000
X=
X= 90 mcgtt/minute

3. A client is being watched for signs of withdrawal as he is being tapered off in his opioids medication. The nurse is aware that the early signs of opioids withdrawal is
A. Increased body temperature
B. Nausea and vomiting
C. Abdominal cramps
D. Profuse sweating
Rationale: D. Diaphoresis is one of the early signs that occur between 6-12 hours of opioids withdrawal. Fever, nausea, and abdominal cramps are considered late signs that occur between 48 and 72 hours. (Mosby’s Prioritization, Delegation, and assignment © 2006, p. 17)

4. A first day post-operative patient who underwent breast lumpectomy and is on a PCA pump complains that the pain control she is receiving is inadequate because there is no signs of relief. What is the first action the nurse needs to take?
A. Recommend the use of non-pharmacological comfort measures
B. Assess the tubing for possible kinks or blockage
C. Call the physician to increase the dose
D. Administer the dose per standing order
Rationale: B. Assessment will enable the nurse to identify the probable reason why the medication is ineffective such a kinks in the tubing that causes the inadequate delivery of the drug. Assessment will guide her on the next step she needs to do. (Mosby’s Prioritization, Delegation, and Assignment © 2006, p. 17)

5. A client who is taking aspirin must be reported immediately to the physician if which of the following laboratory values is observed?
A. Potassium 5.0 mEq/L
B. PT 12 seconds
C. Hct 34%
D. BUN 15 mg/dL
Rationale: C. When a client is taking aspirin monitor for bleeding signs and hematocrit (NV: male 42-52%, female 37-47%). During prolonged therapy, hematocrit, hemoglobin, PT, INR, and renal function must be closely assessed. Aspirin usually causes an increased risk of GI bleeding. (Nursing Drug Handbook © 2003, p.344-345)

6. For a patient with esophageal cancer receiving chemotherapeutic drug vincristine (Oncovin), which of the following side effects should be reported to the physician?
A. Muscle weakness
B. Paresthesia
C. Anorexia
D. Nausea and vomiting
Rationale: B. Paresthesia is one of the side effects of some chemotherapeutic drugs such as vincristine (Oncovin). The physician can modify the dose or discontinue the drug. Muscle weakness, nausea, vomiting, and anorexia are common side effects expected in many chemotherapeutic drugs. (Mosby’s Prioritization, Delegation, and Assignment © 2006, p. 20)

7. Which of the following nursing care is/are appropriate to a patient with leukemia undergoing chemotherapy and taking vincristine (Oncovin)? (Select all that apply)
A. Encourage an increased fluid intake
B. Check for depression of tendon reflex, numbness, or tingling sensation
C. Report any alterations in temperature (fever)
D. Watch for signs of bruising, nose bleeds, or other signs of bleeding tendencies
Rationale: ALL. All of the nursing cares are indicated in a patient taking this chemotherapeutic drug. (Nursing Drug Handbook © 2003, p. 1001)

8. A patient diagnosed with multiple sclerosis was prescribed with baclofen (Lioresal) therapy to reduce muscle spasms as well as urinary self-catheterization every 4 hours. Which instruction should the nurse give to the patient about the medication?
A. “Take it only when symptoms are present”
B. “Take it on an empty stomach, preferably before meal”
C. “Discontinue it if your urine output increases”
D. “Do not abruptly discontinue the drug”
Rationale: D. Baclofen (Lioresal) should not be discontinued abruptly; this may cause hallucinations. The patient should take the drug with food or milk to prevent GI upset. She should not take baclofen only when symptoms are present; to maintain a therapeutic drug level, she must follow the prescribed dosage schedule. Baclofen does not affect the urine output. (Nursing Drug Handbook © 2003, p. 571)

9. A 48-year-old is taking a maintenance therapy of clopidogrel (Plavix) 75 mg PO. The nurse must implement which of the following nursing care for the patient?
A. Instruct patient to drink with juice for more absorption
B. Encourage plenty of fluid intake to prevent constipation
C. Monitor vital signs for sudden elevation of BP
D. Check for signs of bleeding tendencies
Rationale: D. Clopidogrel (Plavix) inhibits platelet aggregation and increases the risk for bleeding. ( NSNA by Stein, 5th ed. p., 60)

10. A 26-year-old office worker is taking a long-term maintenance of olanzapine (Zyprexa) 20 mg PO. Nursing precaution for the patient would include
A. Use cautiously in patient with history of GI bleeding
B. Assisting patient in rising out of bed
C. Frequent checking of vital signs for possible rebound hypertension
D. Monitoring for weight loss
Rationale: B. Olanzapine (Zyprexa) is an antipsychotic medication. Side effects include sedation, weight gain, orthostatic hypotension, blurring of vision, dry mouth, constipation, urinary retention, and photosensitivity reaction. (Nursing Drug Handbook © 2003, p. 483)

11. Which statement of a client taking ibuprofen (Motrin) indicates a need for further teaching?
A. “Ill take my medication with milk”
B. “Ill stop taking gingko biloba”
C. “I will use soft bristle toothbrush”
D. “I will take aspirin one hour before Motrin”
Rationale: D. Ibuprofen (Motrin) is a non-steroidal anti-inflammatory drug. Nursing implication while taking the drug includes taking it with food or milk to decrease GI distress. The drug increases the risk for bleeding so aspirin, gingko biloba and other medication that precipitates bleeding must be stopped and not taken together with the drug. The use of soft bristle toothbrush helps prevent the occurrence of bleeding in the gums. (NSNA by Stein, 5th ed., p. 26)

12. A patient taking St. John’s Worth shows insufficient understanding on the side effects of the drug when he state
A. “Ill take OTC stool softener to prevent constipation”
B. “I feel tired after work”
C. “My heart beats faster than before”
D. “I will use wide brim hat in going to beaches”
Rationale: C. St. John’s Worth is an anti-anxiety, anti-inflammatory, anti-depressive, and sedative agent. Clinical side effects of the drug include photosensitivity, constipation, abdominal cramps, dry mouth, fatigue, dizziness, insomnia, and restlessness. (NSNA by Stein, 5th ed.,p. 109)

13. A patent prescribed with latanoprost (Xalatan) needs further instructions about the medication when he state: (Select all that apply)
A. “The drug may cause an increased brown pigmentation in my iris”
B. “I can use my contact lenses safely after instilling the drug”
C. “It is normal to experience blurring of vision after applying the drug”
D. “Ill administer the drug TID for more effectiveness”
Rationale: BD. Latanoprost (Xalatan) is an ophthalmic solution indicated for increased intraocular pressure in patients with open-angle glaucoma. Expected side effects of the drug include blurring of vision, burning and stinging sensation, increased pigmentation of the iris, excessive tearing, photophobia, and lid discomfort. Patient must be caution not to instill the medication while wearing contact lenses and must wait for at least 15 minutes apart before reinserting the contact lens. The drug is administered once only in the evening because administering the drug more frequently than recommended decreases its IOP lowering effects. (Nursign Drug Handbook © 2003, p. 11210

14. Mrs. A, age 35 is admitted to the local hospital for laparotomy. Post-operatively she is placed on morphine sulfate for pain control. Which o the following symptoms require the nurse to further assesses the patient?
A. Difficulty passing stool
B. Reports of neck and facial itching
C. Feeling of lightheadedness
D. Respiratory rate of 18 breaths per minute
Rationale: B. Morphine sulfate is an opioids that acts on CNS and induces sedation and analgesia. Presence of facial and neck itching must be further assessed because the patent may show a hypersensitivity reaction towards the drug. Respiratory rate is normal and must be documented only. (Nursing Handbook © 2003, p. 386)

15. A man taking orally of finasteride (Proscar). The nurse needs further teaching to the patient when the patient state
A. “The drug will provide an immediate relief”
B. “I will experience a decreased in libido while taking the drug”
C. “I’m having difficulty in ejaculation”
D. “I will have problems in erection”
Rationale: A. Proscar is indicated for symptomatic BPH to improve symptoms and reduce the risk of acute urine retention. Adverse reactions of the drug that must be communicated to the patient include impotence, decreased volume of ejaculation, and decreased libido. The client must also be instructed that the medication will take time before achieving its full therapeutic effect. (NSNA by Stein, 5th ed., p. 53)

16. In order for the nurse to assess the client’s compliance to finasteride (Proscar), she must check while of the following laboratory values?
A. Uric acid precipitate
B. Prostate-specific antigen
C. Low density lipoprotein
D. Alanine aminotransferase
Rationale: B. Finasteride (Proscar) is indicated for BPH. A sustained elevation in prostate-specific antigen levels would indicate noncompliance to the therapy. (Nursing Drug Handbook © 2003, p. 1254)

17. A nurse caring for a patient taking Garamycin for serious infection caused by Pseudomonas aerugenosa would carefully monitor for an elevation of which laboratory values? (Choose all that apply)
A. BUN
B. WBC
C. Creatinine
D. Platelet
E. ALT
F. AST
G. Hemoglobin
H. Bilirubin
Rationale: ACEFH. Garamycin is a member of aminoglycoside. It is widely used to treat serious forms of infection. Adverse reactions of the drug include ototoxicity, hepatotoxicity, and nephrotoxicity causing an elevation in BUN, creatinine, ALT, AST, bilirubin, and LDH levels. The drug may also cause anemia, leucopenia, and thrombocytopenia. (Nursing Drug Handbook © 2003., p. 61)

18. As part of chemotherapy protocol, a female client is to receive cyclophosphaminde (Cytoxan). To prevent the side effects of Cytoxan, she should be instructed to
A. Brush her teeth several times a day
B. Drink at least 2,500 mL of fluids each day
C. Limit food that are high in ascorbic acid in the diet
D. Testing her urine for glucose before meals
Rationale: B. Fluid intake is usually increased to help prevent renal irritation
and hemorrhagic cystitis. Adequate amounts of ascorbic acid are especially
important in the diet. . (Cracking the NCLEX-RN by J.Meyer, 7th ed. © 2003,
p. 236)

19. A client who has schizoaffective disorder has been taking haloperidol (Haldol) for several months. Recently, lithium (Lithane) was added to the medication regime. The nurse in the outpatient program should carefully monitor which of the following due to the combination of these two drugs?
A. Complains of flu-like symptoms, including a temperature of 1000F.
B. Fluctuating levels of consciousness, confusion, and disorientation
C. Increased appetite and a weight gain of 5 pounds
D. Decreased need to sleep and surge of energy
Rationale: B. Potential encephalopathic syndrome can occur with this combination. (Cracking NCLEX-RN by Meyer, 7th ed. p. 579)

20. A client is taking haloperidol (Haldol) 10 mg po hs. In teaching client about self-administration, for which of the following side effects should the nurse emphasize that the client seek to immediate medical attention?
A. Increased urine output
B. Drowsiness
C. Metallic taste and diarrhea
D. Restlessness and muscle spasm
Rationale: D. These are the characteristics of extrapyramidal reactions (akathesia and dystonia) for which the client should be administered a stat dose of anticholinergic medication. Drowsiness may occur when initially taking Haldol . The client should be informed about postural hypotension and taking schedule doses at bedtime. It does not recommend medical intervention. Increased urine output is not associated with Haldol. Metallic taste and diarrhea are not associated with Haldol. (Cracking NCLEX-RN by Meyer, 7th ed. p. 580)

Delegation: 5 items

1. The head nurse is assigning an appropriate client for the LPN/LVN,
who will function under her supervision as the team leader. Which
of the following patients is appropriate to be assigned to the
LPN/LVN?
A. A patient who will undergo surgery and requires a pre-op teaching
B. A patent with HIV who complains of headache and abdominal pain
C. A post-operative toe amputation patient experiencing diabetic neuropathic pain
D. A patient with end stage cancer experiencing pain and refuses to take the medication.
Rationale: C. The patient with toe amputation is a stable patient and needs an ongoing assessment and pain management that are within the scope of an LPN/LPN under the supervision of an RN. The RN should take responsible for the assessment of patent with HIV, pre-op teaching to patient who will undergo surgery, and comprehensive assessment for cancer patient who refuses to take the medication. (Mosby’s Prioritization, Delegation, and Assignment © 2006, p. 17)
2. Which task can be safely delegated to the nursing assistant who is assigned to care for a patient with mouth cancer? (Select all that apply)
A. Observe the patient’s reaction to feedings
B. Assist the patient in performing oral hygiene
C. Encourage expression of grief and anxiety
D. Report temperature elevation
Rationale: B,D. Oral hygiene and reporting changes in vital signs on specific parameters are within the scope of responsibilities of nursing assistants. It is he responsibility of the nurse to observe response to the treatment and help patient deal with loss or anxiety. The nursing assistant can be directed to weight the patient, but should not be expected to know when to initiate that measurement. (Mosby’s Prioritization, Delegation, and Assignment © 2006, p. 19)

3. In assigning nursing care tasks in the unit, which of the following can be appropriately assigned to an LPN/LVN?
A. A patient with severe GI bleeding which requires a blood transfusion
B. A patient who requires antibiotics to control GI bacteria
C. A patient who needs a central line insertion for medication administration
D. A patient who requires a pre-op preparation for bowel resection
Rationale: B. Administering antibiotics is within the scope of practice for LPN/LVNs. Although some states may allow LPN/LVNs to administer blood, in general, blood administration, pre-operative teaching, and central line insertion are responsibilities of RN. (Mosby’s Prioritization, Delegation, and Assignment © 2006, p. 20)

4. A group of health care professionals is preparing a presentation on the prevention and detection of breast cancer. Which task can be delegated to a nurse practitioner?
A. Discuss strategies for reducing the risk factors
B. How to conduct breast self-examination
C. Prepare the visual aids on the warning signs of cancer
D. Formulate a balanced diet to promote healthy living
Rationale: B. The nurse practitioner is often the provider who performs the physical examinations. The RN has the primary responsibility on discussing the risk factors. The LPN/LVN knows the warning signs of cancer and can prepare visual aids for the presentation. Diet planning is the responsibility of nutritionist. (Mosby’s Prioritization, Delegation, and Assignment © 2006, p. 20)

5. Which patients should you assign to the LPN for nursing care
tasks under your supervision? (Choose all that apply)
A. Mr. D, a 69-year-old man with unstable angina who needs
teaching for cardiac catheterization scheduled this morning
B. Ms. J, a 45-year-old woman experiencing chest pain scheduled for a
graded exercise test later today
C. Mr. T, a 67-year-old man with 5-day –old left-sided stroke
D. Ms. S, a 73-year-old woman with a heart disease, a history
of myocardial infarction and mild dementia
E. Ms. M, an 85-year-old woman, newly admitted for long- term care
with decreased urine output, altered level of consciousness, and an
elevated temperature of 99.50F.
F. Mr. P., a 57-year-old man with mild shortness of breath and
chronic emphysema

Rationale: A,C,D,F. it is important to recognize that the RN continues to be accountable for all patient care on this team. Appropriate patient assignments include patients whose conditions are stable and not complex. (Mosby’s Practice Exercises for Med-Surg © 2006, p. 197)